Difference between revisions of "2018 UNM-PNM Statewide High School Mathematics Contest II Problems/Problem 2"

(Included reasoning for solution.)
(Solution)
(Tag: Replaced)
 
(One intermediate revision by the same user not shown)
Line 6: Line 6:
  
 
<math>\{ 1,25,49,73,97 \}</math>
 
<math>\{ 1,25,49,73,97 \}</math>
 
If <math>{a^3}+23</math> is divisible by <math>24</math>, then so is <math>{a^3}-1</math>. <math>{a^3}-1</math> is the same as <math>(a-1)</math><math>({a^2}+a+1)</math>, and it is divisible by <math>24</math>. There are many options: <math>(a-1)</math> is divisible by <math>24</math>, <math>({a^2}+a+1)</math> is divisible by <math>24</math>, <math>(a-1)</math> is divisible by <math>8</math> and <math>({a^2}+a+1)</math> is divisible by <math>3</math>, and <math>(a-1)</math> is divisible by <math>3</math> and <math>({a^2}+a+1)</math> is divisible by <math>8</math>.
 
 
Case 1: <math>(a-1)</math> is divisible by <math>24</math>
 
 
This means that <math>a</math> is congruent to <math>1</math> <math>mod</math> <math>(24)</math>. Satisfying the range, the following integers that satisfy are:
 
 
{<math>1,25,49,73,97</math>}
 
 
Case 2: <math>({a^2}+a+1)</math> is divisible by <math>24</math>
 
 
Or <math>a(a+1)</math> is in the form <math>23</math> <math>mod</math> <math>(24)</math>. This means that <math>a(a+1)</math> can be {<math>23,47,71,95</math>}, in the list, the first three numbers are prime, and The fourth can be factorized into two non-consecutive primes. No results from this case.
 
 
Case 3: <math>(a-1)</math> is divisible by <math>8</math> and <math>({a^2}+a+1)</math> is divisible by <math>3</math>
 
 
<math>a</math> is congruent to <math>1</math> <math>mod</math> <math>(8)</math> or <math>a</math> = <math>8</math> <math>k</math> + <math>1</math> where <math>k</math> is a positive integer. This means that <math>{(8k+1)^2}</math> + <math>8k+1</math> + <math>1</math> = <math>64</math> <math>{k^2}</math> + <math>24</math> <math>k</math> + <math>3</math> which has to be divisible by <math>3</math>. That means so does <math>{k^2}</math>, or <math>k</math> itself is divisible by <math>3</math>. The maximum it can be <math>12</math>, because <math>a<100</math> or <math>a</math> = <math>8k+1</math>. However, for the available values that can be inputted (0,3,6,9,and 12),the same list results from Case 1. No new values.
 
 
Case 4: <math>(a-1)</math> is divisible by <math>3</math> and <math>({a^2}+a+1)</math> is divisible by <math>8</math>
 
 
<math>a</math> is congruent to <math>1</math> <math>mod</math> <math>(3)</math> or <math>a</math> = <math>3</math> <math>k</math> + <math>1</math> where <math>k</math> is a positive integer. This means that <math>{(3k+1)^2}</math> + <math>3k+1</math> + <math>1</math> = <math>9</math> <math>{k^2}</math> + <math>9</math> <math>k</math> + <math>3</math> which has to be divisible by <math>8</math>. That means so does <math>{k^2}</math> + <math>k</math> + <math>3</math>. Checking k modulo eight for all values might result in a value of k which can narrow down search values.
 
 
Sub case 1: k is congruent to -4(mod 8)
 
 
<math>{(-4)^2}</math> + <math>-4</math> + <math>3</math> = 7(mod 8)
 
 
Sub case 2: k is congruent to -3(mod 8)
 
 
<math>{(-3)^2}</math> + <math>-3</math> + <math>3</math> = 1(mod 8)
 
 
Sub case 3: k is congruent to -2(mod 8)
 
 
<math>{(-2)^2}</math> + <math>-2</math> + <math>3</math> = 5(mod 8)
 
 
Sub case 4: k is congruent to -1(mod 8)
 
 
<math>{(-1)^2}</math> + <math>-1</math> + <math>3</math> = 3(mod 8)
 
 
Sub case 5: k is congruent to  0(mod 8)
 
 
<math>{(-0)^2}</math> + <math>-0</math> + <math>3</math> = 3(mod 8)
 
 
Sub case 6: k is congruent to  1(mod 8)
 
 
<math>{(1)^2}</math> + <math>1</math> + <math>3</math> = 5(mod 8)
 
 
Sub case 7: k is congruent to 2(mod 8)
 
 
<math>{(2)^2}</math> + <math>2</math> + <math>3</math> = 1(mod 8)
 
 
Sub case 8: k is congruent to  3(mod 8)
 
 
<math>{(3)^2}</math> + <math>3</math> + <math>3</math> = 7(mod 8)
 
 
In no scenario is <math>{k^2}</math> + <math>k</math> + <math>3</math> divisible by <math>8</math>, and by working backward, neither can <math>({a^2}+a+1)</math>. This means that the list noted in Case 1 are all the numbers possible that satisfy the condition. Our answer is <math>\boxed{\textbf{(1,25,49,73,97)}}</math>
 
  
 
== See also ==
 
== See also ==

Latest revision as of 19:54, 22 August 2023

Problem

Determine all positive integers $a$ such that $a < 100$ and $a^3 + 23$ is divisible by $24$.

Solution

$\{ 1,25,49,73,97 \}$

See also

2018 UNM-PNM Contest II (ProblemsAnswer KeyResources)
Preceded by
Problem 1
Followed by
Problem 3
1 2 3 4 5 6 7 8 9 10
All UNM-PNM Problems and Solutions